Answer (D) is correct . Given that only $600,000 is available and that each project costs $200,000 or more, no more than two projects can be undertaken. Because Projects 3 and 4 have the greatest NPVs, profitability indexes, and IRRs, they are the projects in which the company should invest.
Answer (A) is incorrect because Project 1 has a negative NPV. Answer (B) is incorrect because This answer violates the $600,000 limitation. Answer (C) is incorrect because The combined NPV of Projects 2 and 3 is less than the combined NPV of Projects 3 and?4.
|